Maximum principle - don't understand why RHS is negative.

Click For Summary
SUMMARY

The discussion centers on the maximum principle in the context of a linear inhomogeneous second-order boundary value problem (BVP) defined by the equation -a(x)u''(x) + b(x)u'(x) = f(x) for 0 < x < 1. It is established that if f(x) < 0 for x in [0,1], then the solution u(x) reaches its maximum at the endpoints x = 0 or x = 1. The confusion arises in the second part of the problem regarding the substitution v(x) = u(x) + εeλx, where the right-hand side (RHS) is asserted to be strictly negative due to a(x) > 0, despite uncertainties about the sign of b(x).

PREREQUISITES
  • Understanding of boundary value problems (BVP)
  • Familiarity with linear differential equations
  • Knowledge of the maximum principle in differential equations
  • Basic concepts of function behavior and limits
NEXT STEPS
  • Study the maximum principle for linear differential equations
  • Explore the implications of function substitution in differential equations
  • Investigate the behavior of linear and quadratic functions in boundary value problems
  • Learn about the role of eigenvalues in determining the stability of solutions
USEFUL FOR

Mathematics students, researchers in applied mathematics, and anyone studying differential equations and boundary value problems will benefit from this discussion.

Silversonic
Messages
121
Reaction score
1

Homework Statement



Consider the linear inhomogeneous second order two point BVP;

-a(x)u''(x) + b(x)u'(x) = f(x) for 0 < x < 1

for some functions a, f, b where a(x) > 0 for all x

1) If f(x) < 0 for x = [0,1], show that u(x) attains its maximum value at one of the two end points x = 0, 1. - I've done this, anyone who has done the maximum principle should be able to as well.

2) Substitute v(x) = u(x) + \epsilon e^{\lambda x}, show that if f(x) \leq 0 then u(x) attains its maximum value at one of the end points x = 0,1

The Attempt at a Solution



Question two is what I'm stuck on. If we substitute v(x) into the equation instead we get

[PLAIN]http://img819.imageshack.us/img819/9646/unledmug.jpg

It says the RHS is strictly negative since a(x) > 0. How can we know this, when we don't know what sign b(x) may come out as for any x? It may turn out b(x) is positive and \lambda b is greater than |\lambda^{2 } a| and therefore what is contained within the brackets is positive, and may be bigger than the absolute value of f(x). Making the whole thing positive.

Am I missing something?
 
Last edited by a moderator:
Physics news on Phys.org
for any x, b(x)λ is a linear function in λ.

but a(x)λ2 is quadratic, so for sufficiently large λ, this term will dominate.
 
Question: A clock's minute hand has length 4 and its hour hand has length 3. What is the distance between the tips at the moment when it is increasing most rapidly?(Putnam Exam Question) Answer: Making assumption that both the hands moves at constant angular velocities, the answer is ## \sqrt{7} .## But don't you think this assumption is somewhat doubtful and wrong?

Similar threads

Replies
6
Views
2K
Replies
3
Views
2K
  • · Replies 18 ·
Replies
18
Views
3K
Replies
1
Views
2K
  • · Replies 4 ·
Replies
4
Views
2K
  • · Replies 8 ·
Replies
8
Views
2K
Replies
5
Views
2K
  • · Replies 15 ·
Replies
15
Views
2K
  • · Replies 3 ·
Replies
3
Views
960
Replies
2
Views
2K